Please help me!!! I'm being timed right now do please help me!!!

Please Help Me!!! I'm Being Timed Right Now Do Please Help Me!!!

Answers

Answer 1

Answer:

The answer would be 2. Which means that your answer would be option 1 I believe.

Step-by-step explanation:


Related Questions

Find the x-intercepts of the parabola with
vertex (3,4) and y-intercept (0,-5).
Write your answer in this form: (X1,41), (X2,42).
If necessary, round to the nearest hundredth.
Enter the correct answer.

Answers

the answer is x=-10 because i just did it and got 100

A circle is placed in a square with a side length of 10 mm, as shown below. Find the area of the shaded region.
Use the value 3.14 for it, and do not round your answer. Be sure to include the correct unit in your answer.

Answers

Answer:

21.5

Step-by-step explanation:

The area of the shaded region is 21.5 mm².

What is Area?

Area of a two dimensional shape is the total region which is bounded by the object's shape.

Given is a circle inside a square.

Area of a square = a², where 'a' is the side length.

Here, a = 10 mm

Area of square = 10² = 100 mm²

Area of a circle = π r², where r is the radius of the circle.

Diameter of circle = 10 mm

Radius of the circle = 10/2 = 5 mm

Area of the circle = (3.14) (5²) = 78.5 mm²

Area of the shaded region = Area of square - Area of circle

                                             = 100 mm² - 78.5 mm²

                                             = 21.5 mm²

Hence the area is 21.5 mm².

Learn more about Area here :

https://brainly.com/question/29008139

#SPJ2

Please help me with this assignment step by step answer.

Answers

Answer:

D

Step-by-step explanation:

since they are both outside or just exterior, and they are the opposite, so alternate exterior angles

What are the results of
(3 + 1) + 27 ÷ 9 =

Answers

Answer:

3.4

Step-by-step explanation:

PEMDAS: Parentheses, Exponents, Multiplication and Division (from left to right), Addition and Subtraction (from left to right).

(3+1)=4

4+27=31

31 ÷9=3.4

(hope this helped)

The result of expression(3 + 1) + 27 ÷ 9 = 7 .

What is order of operations?

Order of operations is a set of rules to perform operations in an arithmetic expression.

PEMDAS is an acronym used to mention the order of operations to be followed while solving expressions having multiple operations. PEMDAS stands for P- Parentheses, E- Exponents, M- Multiplication, D- Division, A- Addition, and S- Subtraction.

Given expression

(3 + 1) + 27 ÷ 9

Using order of operations,

Solving Parentheses

= 4 + 27 ÷ 9

Solving Division operation

= 4 + 3

Solving Addition operation

= 7

Hence, 7 is the result of the expression (3 + 1) + 27 ÷ 9 .

Learn more about order of operations here:

https://brainly.com/question/13079176

#SPJ2

Five less then 3 times a number is 3 let x be unknown
number

Answers

Answer:

3x-5=3

Step-by-step explanation:

Answer:

2.6 repeating Or 8/3(same thing in fraction form)

Step-by-step explanation:

So, lets iscolate the different parts of this euqation:

Five less than 3 times a number is 3

Or:

5 less than 3 * x = 3

This is a bit tricky here, but it should be:

(3*x)-5=3

Since order of operations, multipcation comes first.

And remember this is 5 less than the 3*x, so its subtracting 5 from 3*x.

This is all equal to 3.

Now lets solve:

(3*x)-5=3

Solve whats in the parthese first:

3x-5=3

Then get 3x alone:

3x=8

Then get x alone:

x=8/3

Or

2.66 repeating

This is your answer(8/3 or 2.6)

Hope this helps!

defintion of Symmetrical Dot Plot

Answers

Answer: The most typical symmetric histogram or dot plot has the highest vertical column in the center. This shape is often referred to as being a "normal curve" (or normal distribution). ... Skewed Right (positively skewed) - fewer data plots are found to the right of the graph (toward the larger numeric values).

A graph that shows the distribution of a quantitative variable above a number line with small periods, dots, circles, or x's. It puts a quantitative variable against a quantitative variable.

Are ΔSUT and ΔXWV similar? If so, state the reason and the similarity statement.


Question 4 options:

A)

Yes; SSS; ΔSUT ∼ ΔXWV

B)

Yes; SAS; ΔSUT ∼ ΔXWV

C)

Yes; SSS; ΔTSU ∼ ΔXWV

D)

The triangles aren't similar.


Answers

Answer:

Yes (SSS) Triangle SUT is congruent to triangle XWV

Step-by-step explanation:

15/10 = 1.5

9/6 = 1.5

12/8 = 1.5

In ΔSUT and ΔXWV the given sides are in proportion. Therefore, option A is the correct answer.

What are similar triangles?

Two triangles are similar if the angles are the same size or the corresponding sides are in the same ratio. Either of these conditions will prove two triangles are similar.

The given two triangles are ΔSUT and ΔXWV.

From the triangle SUT, SU=10, UT=8 and ST=6

From the triangle XWV, VW=12, XW=15 and VX=9

Now, SU/XW = UT/VW = ST/VX

10/15 = 8/12 = 6/9

2/3 = 2/3 = 2/3

The given sides are in proportion.

So, by SSS similarity ΔSUT∼ΔXWV

Therefore, option A is the correct answer.

To learn more about the similar triangles visit:

https://brainly.com/question/25882965.

#SPJ2

Find the trig ratio. Reduce to
lowest terms.
6
А
8
[?]
sin A
US

Answers

Answer:

[tex]\frac{3}{5}[/tex]

Step-by-step explanation:

GIVEN :-

Measure of side opposite to ∠A (Perpendicular) = 6 unitsMeasure of side adjacent to ∠A (Base) = 8 units

TO FIND :-

sin A

FACTS TO KNOW BEFORE SOLVING :-

[tex]Hypotenuse = \sqrt{(Base)^2 + (Perpendicular)^2}[/tex][tex]\sin A = \frac{Side \: opposite \: to \: A}{Hypotenuse}[/tex]

PROCEDURE :-

Measure of perpendicular of triangle = 6 units

Measure base of triangle = 8 units

Measure of Hypotenuse = [tex]\sqrt{(6)^2 + (8)^2} = \sqrt{36 + 64} = \sqrt{100} = 10 \: units[/tex]

Side opposite to ∠A = Perpendicular of triangle =  6 units

∴ sin A = [tex]\frac{6}{10} = \frac{3}{5}[/tex]

Which of the following is the quotient of the rational expression shown below? 2x+5/3x divided by 2x-1/2x+1

Answers

photo math is a good app for your math questions, i would answer this for you but i’m also doing my school work i wish u luck

What is the lowest whole number for hypotenuse c when using the formula [tex]\sqrt{a^2+b^2}[/tex] and a = b?

Answers

9514 1404 393

Answer:

  0

Step-by-step explanation:

If a=b, you are asking for a whole number c such that ...

  c = √(a² +a²) = a√2

If 'a' is a whole number, the only whole numbers that satisfy this equation are ...

  c = 0 and a = 0.

  0 = 0×√2

The lowest whole number c such that c = √(a²+b²) and a=b=whole number is zero.

__

√2 is irrational, so there cannot be two non-zero whole numbers such that c/a=√2.

_____

Additional comment

If you allow 'a' to be irrational, then you can choose any value of 'c' that you like. Whole numbers begin at 0, so 0 is the lowest possible value of 'c'. If you don't like that one, you can choose c=1, which makes a=(√2)/2 ≈ 0.707, an irrational number. The problem statement here puts no restrictions on the values of 'a' and 'b'.

m∠1=(3x+21)° and m∠3=(4x+9)° what is the value of x

Answers

Value of [tex]x[/tex] as per considering given pair as vertical  opposite angles is equals to [tex]12[/tex] degrees.

What are vertical opposite angles?

" Vertical opposite angles are defined as when two straight line intersect each other , the two non adjacent angles are vertical opposite angles. Measure of vertical opposite angles are always equal."

According to the question,

Given measure of angles,

[tex]m \angle1 = (3x + 21)\°[/tex]

[tex]m \angle3= (4x + 9)\°[/tex]

Consider both the angles as vertically opposite angles we get,

[tex]m \angle1 =m \angle3 \\\implies(3x + 21)\° = (4x + 9)\°\\\\\implies 4x -3x = (21 - 9))\°\\\\\implies x = 12\°[/tex]

Hence, value of [tex]x[/tex] as per considering given pair as vertical  opposite angles is equals to [tex]12[/tex] degrees.

Learn more about vertical opposite angles here

https://brainly.com/question/18045519

#SPJ2

find two numbers if there sum is -11 and there difference is 41

Answers

x = 26. y = -15

Step-by-step explanation:

Let x and y be the two numbers

x + y = 11. #1

x - y = 41. #2

Adding the two equations together, we get

2x = 52 ----> x = 26

Using this value on equation #2, we get

26 - y = 41

or y = -15

“I got a great deal on hamburger, so I bought six pounds. I will use 3/8 of a pound for each hamburger patty (per serving).” How many servings will I get out of the six pounds of hamburger?

Answers

Answer:

16

Step-by-step explanation:

You will get 16 servings out of the six pounds of hamburger.

If you divide the total by the serving (6/(3/8)) you will get 16

How long would it take a cruise ship to travel 156 miles at an average speed of 26 mph

Answers

Answer:

The answer is 6 hours

Step-by-step explanation:

26x6=156

The TSA decides to hold back for inspection 4% of all pieces of carry-on luggage. Assuming selections are independent, for the next 150 pieces of luggage, a) find the exact probability that 7, 8, or 9 will be selected. Interestingly enough, this is a case where two of our approximation methods may be appropriate. b) Using Poisson approximation, recalculate the probability from a), and compare. c) Using normal approximation, recalculate the probability from a), and compare

Answers

Answer:

a) 0.3140

b) 0.3098

c) 0.3447

Step-by-step explanation:

Given data :

p = 4% = 0.04

Assuming selections are independent for next 150 pieces

n = 150

a) Probability that 7,8,9 will be selected

P( 7,8,9 ) = 0.3140

b) probability that 7,8,9 will be selected using Poison approximation

P( 7,8,9 ) = 0.3098

c) using normal approximation to determine the probability from ( a )

P ( 7,8,9 ) = 0.3447

Attached below is the detailed solution

HELP PLEASE SOMEONE:(

Answers

Answer:

C. 5°F

Step-by-step explanation:

- 5°F + 5°F

0°F

1. Write an equation of the line in slope-intercept form that passes through (4.2) and (8.–2).

Answers

Answer:

its (8+4)/(-2-2) so the slope is -3x

Step-by-step explanation:

Sally and Suzie both agree that m∠A + m∠B = 180⁰.

Sally claims ∠A and ∠B are supplementary.

Suzie claims ∠A and ∠B are not supplementary because they are not adjacent.

Who is correct?

Answers

I think Suzie is correct because the angles are separated from each other thus cannot be supplementary

Answer:

Sally is correct in claiming that [tex]\angle A[/tex] and [tex]\angle B[/tex] are supplementary.

Step-by-step explanation:

Supplementary angles are angles that add up to [tex]180^{\circ}[/tex]. Whether they are adjacent or not is irrelevant to whether they are supplementary. Since [tex]70+110=180[/tex], angles [tex]\angle A[/tex] and [tex]\angle B[/tex] are supplementary.

Describe the sequence of transformations from quadrilateral WXYZ to W"X"Y"Z"

Answers

Answer:

[tex]G) (x,y)[/tex] → [tex](x-6,y+5)[/tex]

━◦○◦━◦○◦━◦○◦━◦○◦━◦○◦━

hope it helps..

have a great day!!

Answer:

i got u i have your email

Step-by-step explanation:

83. A student estimated the height of a tree as 47
feet. If the actual height is 53 feet, what is the
percent error rounded to the nearest percent?
(Hint: Compare the estimated height to the actual
height.)
Poviow (SARZA
Page 8

Answers

I believe it’s 89%. I divided the estimated height by the actual height (47/53=0.8867) and then turned it into a percentage (0.8867=89%)

Solvex2
- 4x=21
using the zero product property.

Answers

Answer:

The answer is (x + 3) (x - 7) = 0

Step-by-step explanation:

x² - 4x = 21

x² - 4x - 21 = 0

x² - 7x + 3x - 21 = 0

x(x - 7) + 3(x - 7) = 0

(x + 3) (x - 7) = 0

Thus, The answer is (x + 3) (x - 7) = 0

-TheUnknownScientist

HELP HELP HELP I’ve BEEN STUCK ON THIS QUESTION IVE ASKED LIKE 3 TIMES ALREADY....ILL GIVE BRAINLY!!

Answers

Answer:

y = -3/4x - 3

Step-by-step explanation:

first lets find the slope what is rise over run

to get from (0, -3) to (-4,0) u have to rise 3 and go back 4

the slope would be -3/4

the last part is were it hits the y axis - ( -3) so it be -3

ANSWER PLZZZ I NEED HELP QUICK

Answers

We don’t know, we need the circle graph

Answer:

wheres the circle graph

Step-by-step explanation:

y = a( x-h)^2 + k
h is the _______________________ shift.(+ shifts __________, shifts __________)k is the _______________________ shift. (+ shifts __________, shifts __________)If a is negative, the function is ____________________ across the ____- ________a1represents a vertical ________________________.a0 1represents a vertical ______________________.

can somebody help me fill this out plz

Answers

Can you summarize this?

What is equivalent to 3^-5

Answers

Answer:

The answer is the choose: (D)

[tex] {3}^{ - 5} = \frac{1}{ {3}^{5} } [/tex]

The image
shown at the right contains many triangles.
Describe the different types of triangles found in
the image.

Answers

Answer:

Cna you please put a image, I would like that so much so I can help you?

Step-by-step explanation:

hi there is no picture here to show the image

PLEASE HELP ME WITH THIS

Gabriela is solving the following equation. Her steps are shown:

Gabriella is not sure she has solved the problem correctly. How can you convince Gabriella if she has solved the problem correctly / incorrectly? If she has solved the problem incorrectly, solve it and convince her of the solution

Answers

Answer:

Solution given:

5(9x-8)=4(x+2)

45x-40=4x+8

45x-4x=40+8

41x=48

x=[tex] \frac{48}{41} [/tex]

Gabriella is not correct:

she has done in distribution

5(9x-8)=4(x+2)

45x-40=4x+8 not 45x-8=4x+2.

No, Gabriela is not solving correctly because her second step is wrong.

What is a fraction?

In a fraction the upper part is known as numerator and the lower part is known as denominator.

How to solve the problem?

The problem is as under

(9x-8)/4=(x+2)/5

The above problem is solved as

5(9x-8)=4(x+2)

45x-40=4x+2

41x=42

x=42/41

Hence the value of x is 42/41 therefore the solution of Gabriella is wrong.

Learn more about fraction at https://brainly.com/question/78672

#SPJ2

what decimal is greater than 1.434.

a. 1.443
b. 1.034
c. 1.340
d. 1.433

Answers

Answer: A. D, C, B are all lower

Answer:

the correct answer is A

Step-by-step explanation:

1.443-1.434

= 0.009

How far off the ground is the top of the ladder? URGENT PLS HELP

Answers

The answer is 10.24695. Which is 10.2 for you!

Simplify the variable expression below as much as possible.
3x + 2y + 5-3

Answers

Answer:

3x+2y+2=0

3x=-2y-2

x=-(2y+2)/3

Step-by-step explanation:

Other Questions
Select the correct answer. Simplify the following expression. 3^2/5 * 3^-7/5 The ________________ flows out of a volcano. mantle magma silt lava Please answer the question correctly with factual information for brainliest. Rayann has 4 bracelets, Jennifer has 8 bracelets, June has 3 bracelets, and Tammy has 5 bracelets. They want to redistribute the bracelets so that each girl has the same number of bracelets. If the bracelets are redistributed, how many should each girl get?A balance diagram going from 1 to 9. 1 circle is above 3, 4, 5, and 8.4567 In the 1800s, withwhom did thepeople in the Southwant war? Which reactant is necessary for a combustion reaction?A. WaterO B. HeatO c. Carbon dioxideD. Oxygen 5. Which of the following statements is false? A) The incentives come from owning stock in the company and from compensation that is sensitive to performance. B) The role of the corporate governance system is to mitigate the conflict of interest that results from the combination of ownership and control without unduly burdening managers with the risk of the firm. C) Punishment comes when a board fires a manager for poor performance or fraud, or when, upon failure of the board to act, shareholders or raiders launch control contests to replace the board and management. D) The corporate governance system attempts to align interests by providing incentives for taking the right action and punishments for taking the wrong action. E) None of the above Select all expressions that are equivalent to 4(+2)2+4 A) 64B) 4+22+4 C) 10 D) 428+4 What is the equation of a circle with center (-2,3) and radius 4?A. (x + 2)^2 +(y-3)^2 = 4B. (x-2)^2 +(y +3)^2= 16C. (x + 2)^2 + (y - 3)^2 = 16D. (x+2)^2- (y - 3)^2 - 16 2.reproduction produces offspring with a mixture of the parent's traits.a. Sexualb. AsexualC. Mixtured. Budding What percent of data is less than 20? Please solve these, I thank you my friends Write and solve an equation to find the value of x Here are some pictures of common situations. Talk to each other about how stressful you think each one can be, then, choose the three that you think are the most stressful. It is not necessary to agree with each other.Dialogue about stressful activities (interview, traffic, school, late to work) El siguiente esquema representa un objeto con un volumen de 2cm^3 sumergido en un recipiente que contiene agua. La densidad del agua es 1 g/cm^3 . Si solo la mitad del objeto se encuentra sumergido, el valor de la masa del agua desalojada por el objeto es de: On this day the first all-electronic television image was transmitted. Philo T. Farnsworths invention made it possible to build televisions with no moving parts. What type of influence do you think television has had on our world? What have been some of the advantages? Have there also been some disadvantages? Please write your response in a 7 sentence paragraph. Find the surface area of the prism 12in 7in 5in How did Britain and France respond when Hitler remilitarized the Rhineland?Threatened to invade if he did not remove the troopsBlockaded GermanyAsked the League of Nations to intervene in GermanyDid nothing and appeased Hitler least to greatest -2/5, -2/3, -0.8, -3/4 Does a compromised account on Instamean your banned for a few days????? Yes or no?Answer ASAP What is tourism sector transformation?